Όρια με ολοκληρώματα

Συντονιστές: grigkost, Κοτρώνης Αναστάσιος

Άβαταρ μέλους
mathxl
Δημοσιεύσεις: 6736
Εγγραφή: Τρί Δεκ 23, 2008 3:49 pm
Τοποθεσία: Σιδηρόκαστρο
Επικοινωνία:

Re: Όρια με ολοκληρώματα

#81

Μη αναγνωσμένη δημοσίευση από mathxl » Τετ Ιουν 09, 2010 12:29 pm

27) και φρενάρω για την ώρα :mrgreen:
\displaystyle{\mathop {\lim }\limits_{n \to \infty } \int_0^n l n(1 + {e^{ - x}})dx.}


Ποτε δεν κάνω λάθος! Μια φορά νομιζα πως είχα κάνει, αλλά τελικά έκανα λάθος!
Απ' τα τσακάλια δεν γλυτώνεις μ' ευχές η παρακάλια. Κ. Βάρναλης
Aπέναντι στις αξίες σου να είσαι ανυποχώρητος

Ενεργό μέλος από 23-12-2008 ως και 17-8-2014 (δεν θα απαντήσω σε πμ)
Άβαταρ μέλους
mathxl
Δημοσιεύσεις: 6736
Εγγραφή: Τρί Δεκ 23, 2008 3:49 pm
Τοποθεσία: Σιδηρόκαστρο
Επικοινωνία:

Re: Όρια με ολοκληρώματα

#82

Μη αναγνωσμένη δημοσίευση από mathxl » Τετ Ιουν 09, 2010 12:35 pm

mathxl έγραψε:27) και φρενάρω για την ώρα :mrgreen:
\displaystyle{\mathop {\lim }\limits_{n \to \infty } \int_0^n l n(1 + {e^{ - x}})dx.}
Τι ψεύτης Θεέ μου... 8-)
28) Να δείξετε ότι
\displaystyle{\mathop {\lim }\limits_{n \to \infty } {n^2}\left( {\int_0^1 {\sqrt[n]{{1 + {x^n}}}} dx - 1} \right) = \frac{{{\pi ^2}}}{{12}}.}


Ποτε δεν κάνω λάθος! Μια φορά νομιζα πως είχα κάνει, αλλά τελικά έκανα λάθος!
Απ' τα τσακάλια δεν γλυτώνεις μ' ευχές η παρακάλια. Κ. Βάρναλης
Aπέναντι στις αξίες σου να είσαι ανυποχώρητος

Ενεργό μέλος από 23-12-2008 ως και 17-8-2014 (δεν θα απαντήσω σε πμ)
Άβαταρ μέλους
Κοτρώνης Αναστάσιος
Επιμελητής
Δημοσιεύσεις: 3203
Εγγραφή: Κυρ Φεβ 22, 2009 11:11 pm
Τοποθεσία: Μπροστά στο πισί...
Επικοινωνία:

Re: Όρια με ολοκληρώματα

#83

Μη αναγνωσμένη δημοσίευση από Κοτρώνης Αναστάσιος » Τετ Ιουν 09, 2010 6:31 pm

mathxl έγραψε:24)
Έστω η συνεχής συνάρτηση \displaystyle{f:[0,1] \to (0,\infty )}. Αν α > 0 και \displaystyle{F(\alpha ) = \int_0^1 {f{{(t)}^\alpha }dt} }
να
ι) δείξετε ότι η F είναι παραγωγίσιμη στο (0,1)
ιι) υπολογίσετε το \displaystyle{\mathop {\lim }\limits_{\alpha  \to 0} F{(\alpha )^{\frac{1}{\alpha }}}}
ιιι) υπολογίσετε το \displaystyle{\mathop {\lim }\limits_{\alpha  \to \infty } F{(\alpha )^{\frac{1}{\alpha }}}}
\displaystyle{i)} Η παραγωγισιμότητα της \displaystyle{F(a)} προκύπτει από τον κανόνα του Leibnitz και ισχύει στο \displaystyle{(0,+\infty)}, με \displaystyle{F{'}(a)=\int_{0}^{1}\ln(f(t))e^{a\ln(f(t))}\,dt}

Παρατηρούμε ότι οι \displaystyle{f(t),\,\ln(f(t))} είναι φραγμένες στο \displaystyle{[0,1]}. Από τη συνέχεια τώρα της \displaystyle{e^{x}} στο 0, προκύπτει εύκολα ότι \displaystyle{f^{a}(t)\stackrel{o\mu}{\longrightarrow}1} και \displaystyle{\ln(f(t))f^{a}(t)\stackrel{o\mu}{\longrightarrow}\ln(f(t))} στο \displaystyle{[0,1]} καθώς \displaystyle{a\to0}. \boxed{*}

Τώρα έχουμε :

\displaystyle{ii)} \displaystyle{\lim_{a\to0}\big(F(a)\big)^{1/a}=\lim_{a\to0}e^{(1/a)\ln F(a)}=e^{\lim_{a\to0}(1/a)\ln F(a)}\stackrel{\boxed{*}\Rightarrow0/0}{=}e^{\lim_{a\to0}F{'}(a)/F(a)}\stackrel{\boxed{*}}{=}e^{\int_{0}^{1}\ln(f(t))\,dt}}.
********************************************************************************
Για το iii), δεν ξέρω αν μπορεί να χρησιμοποιηθεί το πρώτο, η λύση μου όμως είναι η εξής :

Θέτω \displaystyle{M=\max_{x\in[0,1]}f(x)}.

Έστω \displaystyle{h>0}. Από τη συνέχεια της \displaystyle{f} βρίσκω \displaystyle{[c,d]\subseteq[0,1]} με \displaystyle{M\geq f(x)\geq M-h>0\,\,\forall x\in[c,d]},

άρα και \displaystyle{f^{a}(x)\geq (M-h)^{a}>0\,\,\forall x\in[c,d]\quad\boxed{*}}.

Επειδή \displaystyle{(d-c)^{1/a}\stackrel{a\to+\infty}{\longrightarrow}1}, βρίσκω και \displaystyle{a_{0}} με \displaystyle{a\geq a_{0}\Rightarrow1-(d-c)^{1/a}\leq\frac{h}{M-h}\Rightarrow(M-h)\Big((d-c)^{1/a}-1\Big)\geq-h\quad\boxed{**}}.

Τώρα

\displaystyle{a\geq a_{0}\Rightarrow\begin{cases}\displaystyle\big(F(a)\big)^{1/a}\leq \Big(\int_{0}^{1}M^{a}\,dx\Big)^{1/a}=M<M+2h \\ \displaystyle\big(F(a)\big)^{1/a}\geq\Big(\int_{0}^{1}(M-h)^{a}\,dx\Big)^{1/a}\geq\Big(\int_{c}^{d}(M-h)^{a}\,dx\Big)^{1/a}=(M-h)(d-c)^{1/h}\stackrel{\boxed{**}}{\geq}M-2h\end{cases}\Rightarrow|\big(F(a)\big)^{1/a}-M|\leq2h},

άρα το όριο είναι \displaystyle{M}.

******************************************************************

Να συμπληρώσω ότι ισχύει και ότι \displaystyle{\lim_{a\to-\infty}\big(F(a)\big)^{1/a}=m}, όπου \displaystyle{m:=\min_{x\in[0,1]}f(x)}.

Απόδειξη :

\displaystyle{\lim_{a\to-\infty}\big(F(a)\big)^{1/a}\stackrel{y=-a}{=}\lim_{y\to+\infty}\Big(\int_{0}^{1}f^{-y}(x)\,dx\Big)^{-1/y}=\lim_{y\to+\infty}\frac{1}{\displaystyle\Big(\int_{0}^{1}\Big(\frac{1}{f(x)}\Big)^{y}\,dx\Big)^{1/y}}\stackrel{ii)}{\longrightarrow}1/\max_{x\in[0,1]}1/f(x)\stackrel{\boxed{*}}{=}\min_{x\in[0,1]}f(x)}.

\displaystyle{\boxed{*}\,\,:} \displaystyle{M_{1/f}:=\max_{x\in[0,1]}1/f(x)\Rightarrow\forall x\in[0,1]: 1/f(x)\leq M_{1/f}\Rightarrow\forall x\in[0,1]: f(x)\geq1/M_{1/f}\Rightarrow1/M_{1/f}=m_{f}} (με τις αντίχτοιχες ισότητες φυσικά να "πιάνονται").

*********************************************************************

Το αντίστοιχο διακριτό ανάλογο των παραπάνω είναι το εξής :

Έστω \displaystyle{a_{1},\ldots,a_{n}} θετικοί πραγματικοί και \displaystyle{p\in\mathbb{R}}.

Αν \displaystyle{p\neq0}, θέτουμε \displaystyle{M_{p}=\Big(\frac{a_{1}^{p}+\cdots+a_{n}^{p}}{n}\Big)^{1/p}} και \displaystyle{M_{0}=(a_{1}\cdots a_{n})^{1/n}}. Δείξτε ότι

\displaystyle{i)} \displaystyle{p<q\Rightarrow M_{p}\leq M_{q}},

\displaystyle{ii)} \displaystyle{\lim_{p\to+\infty}M_{p}=\max\{a_{1},\ldots,a_{n}\}}

\displaystyle{iii)} \displaystyle{\lim_{p\to-\infty}M_{p}=\min\{a_{1},\ldots,a_{n}\}} και

\displaystyle{iv)} \displaystyle{\lim_{p\searrow 0}M_{p}=M_{0}}

(Νεγρεπόντης ΙΙα σ.155 ασκ 21.100)
τελευταία επεξεργασία από Κοτρώνης Αναστάσιος σε Πέμ Ιουν 10, 2010 1:27 pm, έχει επεξεργασθεί 3 φορές συνολικά.


Εσύ....; Θα γίνεις κανίβαλος....;
Άβαταρ μέλους
mathxl
Δημοσιεύσεις: 6736
Εγγραφή: Τρί Δεκ 23, 2008 3:49 pm
Τοποθεσία: Σιδηρόκαστρο
Επικοινωνία:

Re: Όρια με ολοκληρώματα

#84

Μη αναγνωσμένη δημοσίευση από mathxl » Τετ Ιουν 09, 2010 6:39 pm

Η παραπάνω άσκηση είναι από \displaystyle{{\rm{Admitere SNSB}},{\rm{ 2}}00{\rm{6}}} και δεν είχε λύση από εκεί που την πήρα


Ποτε δεν κάνω λάθος! Μια φορά νομιζα πως είχα κάνει, αλλά τελικά έκανα λάθος!
Απ' τα τσακάλια δεν γλυτώνεις μ' ευχές η παρακάλια. Κ. Βάρναλης
Aπέναντι στις αξίες σου να είσαι ανυποχώρητος

Ενεργό μέλος από 23-12-2008 ως και 17-8-2014 (δεν θα απαντήσω σε πμ)
Άβαταρ μέλους
Κοτρώνης Αναστάσιος
Επιμελητής
Δημοσιεύσεις: 3203
Εγγραφή: Κυρ Φεβ 22, 2009 11:11 pm
Τοποθεσία: Μπροστά στο πισί...
Επικοινωνία:

Re: Όρια με ολοκληρώματα

#85

Μη αναγνωσμένη δημοσίευση από Κοτρώνης Αναστάσιος » Τετ Ιουν 09, 2010 6:45 pm

Ας δούμε και μια παρόμοια με την 24)

29) Έστω \displaystyle{f[a,b]\to\mathbb{R}} (\displaystyle{\inf f>0} για τα \displaystyle{ii),iii)}) ολοκληρώσιμη.

Θέτουμε \displaystyle{f_{kn}=f(a+k\delta_{n})} με \displaystyle{\delta_{n}:=\frac{b-a}{n}}. Τότε :

\displaystyle{i)} \displaystyle{\lim_{n\to+\infty}\frac{f_{1n}+\cdots+f_{nn}}{n}=\frac{1}{b-a}\int_{a}^{b}f(x)\,dx},

\displaystyle{ii)} \displaystyle{\lim_{n\to+\infty}\sqrt[n]{f_{1n}\cdots f_{nn}}=e^{\frac{1}{b-a}\int_{a}^{b}\ln(f(x))\,dx}}, και

\displaystyle{iii)} \displaystyle{\lim_{n\to+\infty}\frac{n}{1/f_{1k}+\cdots+1/f_{1n}}=\frac{b-a}{\int_{a}^{b}1/f(x)\,dx}}

Υ.Γ.: Γενικεύεται και στα γενικευμένα ολοκληρώματα
τελευταία επεξεργασία από Κοτρώνης Αναστάσιος σε Τρί Οκτ 05, 2010 1:28 pm, έχει επεξεργασθεί 3 φορές συνολικά.


Εσύ....; Θα γίνεις κανίβαλος....;
Άβαταρ μέλους
Κοτρώνης Αναστάσιος
Επιμελητής
Δημοσιεύσεις: 3203
Εγγραφή: Κυρ Φεβ 22, 2009 11:11 pm
Τοποθεσία: Μπροστά στο πισί...
Επικοινωνία:

Re: Όρια με ολοκληρώματα

#86

Μη αναγνωσμένη δημοσίευση από Κοτρώνης Αναστάσιος » Τετ Ιουν 09, 2010 6:52 pm

Μια γενίκευση της \displaystyle{24)} \displaystyle{iii)} υπάρχει εδώ με τη μόνη διαφορά ότι εδώ η μεταβλητή ως προς την οποίια παίρνουμε όριο είναι συνεχής
τελευταία επεξεργασία από Κοτρώνης Αναστάσιος σε Τρί Οκτ 05, 2010 1:28 pm, έχει επεξεργασθεί 1 φορά συνολικά.


Εσύ....; Θα γίνεις κανίβαλος....;
Άβαταρ μέλους
mathxl
Δημοσιεύσεις: 6736
Εγγραφή: Τρί Δεκ 23, 2008 3:49 pm
Τοποθεσία: Σιδηρόκαστρο
Επικοινωνία:

Re: Όρια με ολοκληρώματα

#87

Μη αναγνωσμένη δημοσίευση από mathxl » Τετ Ιουν 09, 2010 11:39 pm

Τάσο έχω ήδη δώσει μία για 28 , αν βάλουμε 29 την δική σου τότε η ακόλουθη είναι η
30) Να προσδιορίσετε τον πραγματικ'ό αριθμό α ώστε \displaystyle{L \in \left( {0, + \infty } \right)}
με \displaystyle{L = \mathop {\lim }\limits_{n \to  + \infty } {n^\alpha }\int\limits_0^1 {{{\sin }^{2n}}\frac{{\pi x}}{2}dx} }
Για την τιμή του α που βρήκατε να υπολογίσετε και το όριο L


Ποτε δεν κάνω λάθος! Μια φορά νομιζα πως είχα κάνει, αλλά τελικά έκανα λάθος!
Απ' τα τσακάλια δεν γλυτώνεις μ' ευχές η παρακάλια. Κ. Βάρναλης
Aπέναντι στις αξίες σου να είσαι ανυποχώρητος

Ενεργό μέλος από 23-12-2008 ως και 17-8-2014 (δεν θα απαντήσω σε πμ)
Άβαταρ μέλους
Κοτρώνης Αναστάσιος
Επιμελητής
Δημοσιεύσεις: 3203
Εγγραφή: Κυρ Φεβ 22, 2009 11:11 pm
Τοποθεσία: Μπροστά στο πισί...
Επικοινωνία:

Re: Όρια με ολοκληρώματα

#88

Μη αναγνωσμένη δημοσίευση από Κοτρώνης Αναστάσιος » Πέμ Ιουν 10, 2010 10:36 am

mathxl έγραψε:27) και φρενάρω για την ώρα :mrgreen:
\displaystyle{\mathop {\lim }\limits_{n \to \infty } \int_0^n l n(1 + {e^{ - x}})dx.}
\displaystyle{\int_0^n l n(1 + {e^{ - x}})dx\stackrel{x=ny}{=}\int_{0}^{1}n\ln\Big(1+\frac{1}{e^{ny}}\Big)\,dy=\int_{0}^{1}n\sum_{k=1}^{+\infty}(-1)^{k-1}\frac{1}{k e^{nky}}=\sum_{k=1}^{+\infty}n\frac{(-1)^{k-1}}{k}\int_{0}^{1}e^{-nky}\,dy=\sum_{k=1}^{+\infty}(-1)^{k-1}\frac{1-e^{-nk}}{k^{2}}=}

\displaystyle{\sum_{k=1}^{+\infty}(-1)^{k-1}\frac{1}{k^2}-\sum_{k=1}^{+\infty}(-1)^{k-1}\frac{e^{-kn}}{k^{2}}=\frac{\pi^{2}}{12}-\sum_{k=1}^{+\infty}(-1)^{k-1}\frac{e^{-kn}}{k^{2}}\to\frac{\pi^{2}}{12}-0}


Εσύ....; Θα γίνεις κανίβαλος....;
Άβαταρ μέλους
mathxl
Δημοσιεύσεις: 6736
Εγγραφή: Τρί Δεκ 23, 2008 3:49 pm
Τοποθεσία: Σιδηρόκαστρο
Επικοινωνία:

Re: Όρια με ολοκληρώματα

#89

Μη αναγνωσμένη δημοσίευση από mathxl » Πέμ Ιουν 10, 2010 11:00 am

Την ίδια λύση έχω δει και εγώ η οποία βασίζεται στην σειρά \displaystyle{ln(1 + x) = \sum\limits_{n \ge 0} (  - 1{)^n}\frac{{{x^{n + 1}}}}{{n + 1}}}
χωρίς αλλαγή μεταβλητής

\displaystyle{\int\limits_0^n l n(1 + {e^{ - x}}) = \int\limits_0^n {\sum\limits_{k \ge 0} ( }  - 1{)^k}\frac{{{e^{ - x(k + 1)}}}}{{k + 1}} = }

\displaystyle{ = \sum\limits_{k \ge 0} {\frac{{{{( - 1)}^k}}}{{k + 1}}} \int\limits_0^n {{e^{ - x(k + 1)}}}  =   \sum\limits_{k \ge 0} {\frac{{{{( - 1)}^{k + 1}}}}{{{{(k + 1)}^2}}}} ({e^{ - n(k + 1)}} - 1) = }

\displaystyle{ = {e^{ - n}}\sum\limits_{k \ge 0} {\frac{{{{( - 1)}^{k + 1}}}}{{{{(k + 1)}^2}}}} {e^{ - nk}} - \sum\limits_{k \ge 0} {\frac{{{{( - 1)}^k}}}{{{{(k + 1)}^2}}}}  = {e^{ - n}}C + \frac{{{\pi ^2}}}{{12}} \to \frac{{{\pi ^2}}}{{12}}}


Με τέτοιο τρόπο βγαίνει \displaystyle{\int\limits_0^n l n(1 + {e^{ - x}})dx = \int\limits_0^n {\int_0^{{e^{ - x}}} {\frac{1}{{1 + u}}\,du} } dx}
;;; (Αλλαγή σειράς ολοκλήρωσης);


Ποτε δεν κάνω λάθος! Μια φορά νομιζα πως είχα κάνει, αλλά τελικά έκανα λάθος!
Απ' τα τσακάλια δεν γλυτώνεις μ' ευχές η παρακάλια. Κ. Βάρναλης
Aπέναντι στις αξίες σου να είσαι ανυποχώρητος

Ενεργό μέλος από 23-12-2008 ως και 17-8-2014 (δεν θα απαντήσω σε πμ)
Άβαταρ μέλους
Κοτρώνης Αναστάσιος
Επιμελητής
Δημοσιεύσεις: 3203
Εγγραφή: Κυρ Φεβ 22, 2009 11:11 pm
Τοποθεσία: Μπροστά στο πισί...
Επικοινωνία:

Re: Όρια με ολοκληρώματα

#90

Μη αναγνωσμένη δημοσίευση από Κοτρώνης Αναστάσιος » Πέμ Ιουν 10, 2010 5:07 pm

mathxl έγραψε:Τι ψεύτης Θεέ μου... 8-)
28) Να δείξετε ότι
\displaystyle{I=\mathop {\lim }\limits_{n \to \infty } {n^2}\left( {\int_0^1 {\sqrt[n]{{1 + {x^n}}}} dx - 1} \right) = \frac{{{\pi ^2}}}{{12}}.}
Πάρα πολύ μου άρεσε αυτό!

Λοιπόν:

\displaystyle{I=\lim_{n\to+\infty}n^{2}\int_{0}^{1}(1+x^{n})^{1/n}-1\,dx=\lim_{n\to+\infty}n^{2}\int_{0}^{1}\sum_{k=1}^{+\infty}\binom{1/n}{k}x^{nk}\,dx=\lim_{n\to+\infty}n^{2}\sum_{k=1}^{+\infty}\binom{1/n}{k}\int_{0}^{1}x^{nk}\,dx=}

\displaystyle{\lim_{n\to+\infty}n^{2}\sum_{k=1}^{+\infty}\binom{1/n}{k}\frac{1}{nk+1}=\lim_{n\to+\infty}\sum_{k=1}^{+\infty}\frac{n^{2}}{nk+1}\frac{\frac{1}{n}\Big(\frac{1}{n}-1\Big)\Big(\frac{1}{n}-2\Big)\cdots\Big(\frac{1}{n}-(k-1)\Big)}{k!}=}

\displaystyle{\lim_{n\to+\infty}\left(\lim_{i\to+\infty}\sum_{k=1}^{i}(-1)^{k-1}\frac{(n-1)(2n-1)\cdots\big((k-1)n-1\big)}{n^{k-2}k!(nk+1)}\right)}

\displaystyle{\lim_{n\to+\infty}\left(\lim_{i\to+\infty}\sum_{k=1}^{i}\frac{(-1)^{k-1}(k-1)!n^{k-1}+a_{k-2}n^{k-2}+\cdots+(-1)^{k-1}}{(k-1)!k^{2}n^{k-1}+k!n^{k-2}}\right)=} (εναλλαγή ορίων)

\displaystyle{\lim_{i\to+\infty}\left(\lim_{n\to+\infty}\sum_{k=1}^{i}\frac{(-1)^{k-1}(k-1)!n^{k-1}+a_{k-2}n^{k-2}+\cdots+(-1)^{k-1}}{(k-1)!k^{2}n^{k-1}+k!n^{k-2}}\right)=} (για σταθερό k έχουμε πηλίκο πολυωνύμων του \displaystyle{n} ίδιου βαθμού \displaystyle{k-1})

\displaystyle{\lim_{i\to+\infty}\sum_{k=1}^{i}(-1)^{k-1}\frac{1}{k^{2}}=\frac{\pi^{2}}{12}},

Με την εναλλαγή των ορίων να δικαιολογείται λόγω απόλυτης σύγκλισης.


Εσύ....; Θα γίνεις κανίβαλος....;
Άβαταρ μέλους
mathxl
Δημοσιεύσεις: 6736
Εγγραφή: Τρί Δεκ 23, 2008 3:49 pm
Τοποθεσία: Σιδηρόκαστρο
Επικοινωνία:

Re: Όρια με ολοκληρώματα

#91

Μη αναγνωσμένη δημοσίευση από mathxl » Πέμ Ιουν 10, 2010 5:16 pm

Τάσο το θέμα είναι από Vojtech Jarnik, 2002, Categoria II και είνα αναπάντητο από εκεί που το πήρα


Ποτε δεν κάνω λάθος! Μια φορά νομιζα πως είχα κάνει, αλλά τελικά έκανα λάθος!
Απ' τα τσακάλια δεν γλυτώνεις μ' ευχές η παρακάλια. Κ. Βάρναλης
Aπέναντι στις αξίες σου να είσαι ανυποχώρητος

Ενεργό μέλος από 23-12-2008 ως και 17-8-2014 (δεν θα απαντήσω σε πμ)
Άβαταρ μέλους
Κοτρώνης Αναστάσιος
Επιμελητής
Δημοσιεύσεις: 3203
Εγγραφή: Κυρ Φεβ 22, 2009 11:11 pm
Τοποθεσία: Μπροστά στο πισί...
Επικοινωνία:

Re: Όρια με ολοκληρώματα

#92

Μη αναγνωσμένη δημοσίευση από Κοτρώνης Αναστάσιος » Πέμ Ιουν 10, 2010 5:17 pm

mathxl έγραψε:Τάσο το θέμα είναι από Vojtech Jarnik, 2002, Categoria II και είνα αναπάντητο από εκεί που το πήρα
Αυτές οι ονομασίες που δίνεις είναι forums;


Εσύ....; Θα γίνεις κανίβαλος....;
Άβαταρ μέλους
mathxl
Δημοσιεύσεις: 6736
Εγγραφή: Τρί Δεκ 23, 2008 3:49 pm
Τοποθεσία: Σιδηρόκαστρο
Επικοινωνία:

Re: Όρια με ολοκληρώματα

#93

Μη αναγνωσμένη δημοσίευση από mathxl » Πέμ Ιουν 10, 2010 5:18 pm

Διαγωνισμοί πρέπει να είναι που βρίσκω από φόρουμ


Ποτε δεν κάνω λάθος! Μια φορά νομιζα πως είχα κάνει, αλλά τελικά έκανα λάθος!
Απ' τα τσακάλια δεν γλυτώνεις μ' ευχές η παρακάλια. Κ. Βάρναλης
Aπέναντι στις αξίες σου να είσαι ανυποχώρητος

Ενεργό μέλος από 23-12-2008 ως και 17-8-2014 (δεν θα απαντήσω σε πμ)
Άβαταρ μέλους
mathxl
Δημοσιεύσεις: 6736
Εγγραφή: Τρί Δεκ 23, 2008 3:49 pm
Τοποθεσία: Σιδηρόκαστρο
Επικοινωνία:

Re: Όρια με ολοκληρώματα

#94

Μη αναγνωσμένη δημοσίευση από mathxl » Παρ Ιουν 11, 2010 2:02 pm

31) Ας δούμε (ένα γράμμα πιο αριστερά στο πληκτρολόγιο Φωτεινή :mrgreen: -όπως καταλαβαίνεις, δεν κοιτώ τι γράφω :lol: ) και αυτό το όριο
\displaystyle{ \mathop{\lim }\limits_{x\to+\infty }\int\limits_{0}^{x}{\sin\left({{t^{2}}}\right)dt} }


Ποτε δεν κάνω λάθος! Μια φορά νομιζα πως είχα κάνει, αλλά τελικά έκανα λάθος!
Απ' τα τσακάλια δεν γλυτώνεις μ' ευχές η παρακάλια. Κ. Βάρναλης
Aπέναντι στις αξίες σου να είσαι ανυποχώρητος

Ενεργό μέλος από 23-12-2008 ως και 17-8-2014 (δεν θα απαντήσω σε πμ)
Άβαταρ μέλους
silouan
Επιμελητής
Δημοσιεύσεις: 1398
Εγγραφή: Τρί Ιαν 27, 2009 10:52 pm

Re: Όρια με ολοκληρώματα

#95

Μη αναγνωσμένη δημοσίευση από silouan » Παρ Ιουν 11, 2010 3:06 pm

Είναι τα λεγόμενα Fresnel integrals !
To αποτέλεσμα είναι \displaystyle{\frac{1}{2}\sqrt{\frac{\pi}{2}}}
Από ότι θυμάμαι όμως δεν είναι και πολύ εύκολο στον υπολογισμό!


Σιλουανός Μπραζιτίκος
Άβαταρ μέλους
chris_gatos
Επιμελητής
Δημοσιεύσεις: 6962
Εγγραφή: Κυρ Δεκ 21, 2008 9:03 pm
Τοποθεσία: Ανθούπολη

Re: Όρια με ολοκληρώματα

#96

Μη αναγνωσμένη δημοσίευση από chris_gatos » Παρ Ιουν 11, 2010 3:11 pm



Χρήστος Κυριαζής
Ωmega Man
Δημοσιεύσεις: 1264
Εγγραφή: Παρ Ιουν 05, 2009 8:17 am

Re: Όρια με ολοκληρώματα

#97

Μη αναγνωσμένη δημοσίευση από Ωmega Man » Παρ Ιουν 11, 2010 3:24 pm

Από ότι θυμάμαι όμως δεν είναι και πολύ εύκολο στον υπολογισμό!
Δες και εδώ.


What's wrong with a Greek in Hamburg?
Άβαταρ μέλους
mathxl
Δημοσιεύσεις: 6736
Εγγραφή: Τρί Δεκ 23, 2008 3:49 pm
Τοποθεσία: Σιδηρόκαστρο
Επικοινωνία:

Re: Όρια με ολοκληρώματα

#98

Μη αναγνωσμένη δημοσίευση από mathxl » Παρ Ιουν 11, 2010 4:00 pm

Ωmega Man έγραψε:
Από ότι θυμάμαι όμως δεν είναι και πολύ εύκολο στον υπολογισμό!
Δες και εδώ.
Αυτό ήθελα Γιώργο, ευχαριστώ
Δίνω και μια άλλη λύση που βρήκα με Γ και διπλό ολοκλήρωμα εδώ http://www.artofproblemsolving.com/Foru ... +integrals
ο κόλπο ακόμη πιο γρήγορα και όμορφα εδώ http://www.mathhelpforum.com/math-help/ ... ost44.html

:shock: Τάσο έφαγες χαλασμένο φαγητό και πήρες χρώμα σαν μελιτζάνα? :mrgreen:
Καλορίζικοι οι επιμελητές μας :)


Ποτε δεν κάνω λάθος! Μια φορά νομιζα πως είχα κάνει, αλλά τελικά έκανα λάθος!
Απ' τα τσακάλια δεν γλυτώνεις μ' ευχές η παρακάλια. Κ. Βάρναλης
Aπέναντι στις αξίες σου να είσαι ανυποχώρητος

Ενεργό μέλος από 23-12-2008 ως και 17-8-2014 (δεν θα απαντήσω σε πμ)
Άβαταρ μέλους
Κοτρώνης Αναστάσιος
Επιμελητής
Δημοσιεύσεις: 3203
Εγγραφή: Κυρ Φεβ 22, 2009 11:11 pm
Τοποθεσία: Μπροστά στο πισί...
Επικοινωνία:

Re: Όρια με ολοκληρώματα

#99

Μη αναγνωσμένη δημοσίευση από Κοτρώνης Αναστάσιος » Παρ Ιουν 11, 2010 6:10 pm

Με διπλό ολοκλήρωμα και εναλλαγή σειράς άθροισης (χωρίς μιγαδική ανάλυση) υπάρχει και στο βιβλίο του Nowak Problems in mathematical analysis III p.191
τελευταία επεξεργασία από Κοτρώνης Αναστάσιος σε Παρ Ιουν 11, 2010 7:03 pm, έχει επεξεργασθεί 1 φορά συνολικά.


Εσύ....; Θα γίνεις κανίβαλος....;
Άβαταρ μέλους
Κοτρώνης Αναστάσιος
Επιμελητής
Δημοσιεύσεις: 3203
Εγγραφή: Κυρ Φεβ 22, 2009 11:11 pm
Τοποθεσία: Μπροστά στο πισί...
Επικοινωνία:

Re: Όρια με ολοκληρώματα

#100

Μη αναγνωσμένη δημοσίευση από Κοτρώνης Αναστάσιος » Παρ Ιουν 11, 2010 6:46 pm

mathxl έγραψε:28) Να δείξετε ότι
\displaystyle{\mathop {\lim }\limits_{n \to \infty } {n^2}\left( {\int_0^1 {\sqrt[n]{{1 + {x^n}}}} dx - 1} \right) = \frac{{{\pi ^2}}}{{12}}.}
Είναι συγκεκριμένα το πρόβλημα 4 του 2002 της κατηγορίας 2 εδώ.


Εσύ....; Θα γίνεις κανίβαλος....;
Απάντηση

Επιστροφή σε “ΑΝΑΛΥΣΗ”

Μέλη σε σύνδεση

Μέλη σε αυτήν τη Δ. Συζήτηση: Δεν υπάρχουν εγγεγραμμένα μέλη και 11 επισκέπτες